+1 Daumen
570 Aufrufe

Mir fällt es schwer, diese Gleichung auf beiden Seiten zu integrieren:

\( \frac{\mathrm{d} v}{\mathrm{~d} t}=a(v)=a_{0}-k v \)

\( \rightarrow \frac{\mathrm{d} v}{a_{0}-k v}=\mathrm{d} t \)


Die Gleichung stammt aus der Mechanik I und es wurde eine Trennung der veränderlichen durchgeführt, aber ich verstehe nicht wie ich links integrieren muss.

Ich dachte, die linke Seite wäre etwas wie z.b 5/(2x +4) dann wäre dies integriert 5·ln|2x+4|

und es käme bei der Aufgabe: v*ln|ao -kv| = t heraus.

Avatar von

1 Antwort

0 Daumen
 
Beste Antwort

$$\frac{dv}{a_0-kv}=dt \Rightarrow \int \frac{1}{a_0-kv}dv=\int dt \Rightarrow -\frac{1}{k}\ln |a_0-kv|=t+c$$

Avatar von 6,9 k

Super danke für deine Antwort!
Ich glaub ich hab da einen schwerwiegenden fehler bei mir,.. ich erkenne nämlich nicht die integrations Variablen scheinbar nicht .

also auf der linken Seite ist es t, das ist klar aber wieso ist es auf der rechten Seite nicht v  statt k? Ich mein da steht doch dv ... also es wäre doch sowas wie :

$$\frac { 1 }{ (3+4x) } dx\quad =\quad dy\quad \quad =>\quad \frac { 1 }{ 4 } ln|3+4x|=y$$


oder nicht ?

An welche Stelle meinst du dass es v statt k sein sollte? 

Meinst dass es nach der Integration $$\int \frac{1}{a_0-kv}dv=\int dt$$ folgenderweise sein sollte $$-\frac{1}{v} \ln |a_0-kv|=t+c$$ ?

Bei deinem Beispiel integrierst du nach x und hast dann vor den Logarithmus die Konstante $$\frac{1}{4}$$ da im Logarithmus das x mit 4 multiplizierst ist.

Hier anstatt 4 haben wir -k.

Genau so :)
woran hast du erkannt das du nach k integrierst statt nach v? Ich meine k ist scheinbar richtig, allerdings verstehe ich nicht woran man das sehen sollte , denn  am Anfang in der Aufgabe stand ja  ... a(v)= a0 -kv
oh nein, es hat gerade klick gemacht! maiem vielen vielen Dank für die Zeit und Mühe, ich habs verstanden :)

Man integriert nach "v". Man hat ja "dv".

Das "-k" ist eine Konstante mit der das x multipliziert ist, wie in deinem Beispiel das "4".


Wenn du folgende Funktion differentierst $$f(v)=\ln |a_0-kv|$$ bekommst du $$f'(v)=\frac{-k}{a_0-kv}$$ aber im Integral hast du $$\frac{1}{a_0-kv}$$ also fehlt das -k.


Also $$\int \frac{1}{a_0-kv}dv=\frac{1}{-k} \int \frac{-k}{a_0-kv}dv=\frac{-1}{k} \int (\ln |a_0-kv|)'dv=\frac{-1}{k}\ln |a_0-kv|$$



Ein anderes Problem?

Stell deine Frage

Willkommen bei der Mathelounge! Stell deine Frage einfach und kostenlos

x
Made by a lovely community